g A company's most recent balance sheet reported total assets of $1.9 million, total liabilities of $0.8 million, and total equity of $1.1 million. Its Debt to equity ratio is: Group of answer choices

Answers

Answer 1

Answer:

0.73

Explanation:

Debt to equity ratio is calculated as Total debt / Total equity

= $0.8 million / $1.1 million

= 0.73

Therefore, debt to equity ratio is 0.73


Related Questions

When gasoline gallons are priced in terms of number of seashells, seashells serve as: Group of answer choices

Answers

Answer:

Unit of account

Explanation:

Money serves three functions :

1. Unit of account : money serves the function of determining the value of a good or service. It is usually assumed that goods that are more highly priced are more valuable that goods that have lower prices

2. Medium of exchange : goods and services can be exchanged for money. For example, if I want to buy a gallon of gasoline and pay 4 seashells, money has served as a medium of exchange.

3. store of value: money can be saved, retrieved and exchanged sometimes in the future

You purchased a share of stock for $120. One year later you received $1.82 as a dividend and sold the share for $136. What was your holding-period return

Answers

Answer:

Holding period return =14.85 %

Explanation:

The return on stock is the sum of the dividends earned and capital gains made during the holding period of the investment.

Dividend is the proportion of the profit made by a company which is paid to shareholders.  

Capital gains is another type of the return made on an equity investment as a result of increase in the value of the shares. It is difference between the cost of the share and the value at the time of disposal.

Therefore, we can can compute the return on the investment as follows:

Holding period return = (Dividend + capital gain)/Begin Price of stock × 100  

Dividend = $1.82

Capital gains= 136 - 120 = 16

Total dollar return on Investment = 1.82 + 16= $ 17.82

                                      = 17.82/120 × 100 = 14.85 %

Holding period return =14.85 %

The calculation of WACC involves calculating the weighted average of the required rates of return on debt and equity, where the weights equal the percentage of each type of financing in the firm's overall capital structure.
_________ is the symbol that represents the cost of preferred stock in the weighted average cost of capital (WACC) equation.
Bryant Co. has $2.3 million of debt, $1.5 million of preferred stock, and $1.8 million of common equity. What would be its weight on common equity?
A. 0.32
B. 0.24
C. 0.22
D. 0.30

Answers

Answer:

Option A is the correct answer

Weight of equity =0.32

Explanation:

Weighted average cost of capital is the average cost of all of the long-term types of finance used by a company weighted according to the that amount of finance used in relation to the total pool of fund.

The weight is the market value of nominal value of the source of fund as a proportion of the total capital funds.

Total capital funds = Debt funds + Preferred Funds + Equity funds

                              = ($2.3 + $1.5 + $1.8 ) million  = $5.6  million

Weight of equity = Equity capital/Total capital funds

                           = 1.8/5.6 =0.32

Weight of equity =0.32

You are aware that your neighbor trades stocks based on confidential information he overhears at his workplace. This information is not available to the general public. This neighbor continually brags to you about the profits he earns on these trades. Given this, you would tend to argue that the financial markets are at best _____ form efficient.

Answers

Answer:

Semi-strong Form Efficient.

Explanation:

There are three levels of market efficiency as weak, semi-strong and strong.

In a semi-strong form efficient market, the stock prices change independently of the previous return points and the current information so it is not possible to predict the future stock prices.

The example given in the question, which states that the neighbor has non-public information, can be classified as a semi-strong form efficient market.

I hope this answer helps.

Based on your case knowledge, to what extent do you agree or disagree with the following statement - "Kay Whitmore - Kodak CEO, had an understanding of Kodak's potential in the PC market. This was illustrated by her strong engagement with Bill Gates and Microsoft."
1. Strongly Agree
2. Mildly Agree
3. Neither Agree nor Disagree
4. MIldly Disagree
5. Strongly Disagree
6. Not Applicable

Answers

Answer:

3. Neither Agree nor Disagree

Explanation:

The reason was that the Kay Whitmore's engagement with Bill Gates and Microsoft has not much impacts on the potential of Kodak's products to exploit additional opportunities in Microsoft hence statement in consideration is not a one side argument as it is doubtful position.

So I am neither agreeing nor disagreeing with the statement hence the option 3 is correct here.

Prepare the journal entry to record Jevonte Company’s issuance of 35,000 shares of its common stock assuming the shares have a: $3 par value and sell for $22 cash per share. $3 stated value and sell for $22 cash per share.

Answers

Answer: Please see answer in explanation column

Explanation:

a)journal entry to record Jevonte Company’s issuance at $3 par value and $22 cash per share

Account                                            Debit                        Credit

Cash(35,000 x $22)                       $770,000

Common stock, $3 par value(35,000 x 3)                       $105, 000

Paid-in captial in excess of par value, common stock

($770,000  - $105, 000 )                                                      $665,000

b)journal entry to record Jevonte Company’s issuance at $3 stated  value and $22 cash per share

Account                                            Debit                        Credit

Cash  (35,000 x $22)                    $770,000

Common stock, $3 stated value (35,000 x 3)                 $105, 000

Paid-in captial in excess of stated value, common stock

($770,000  - $105, 000 )                                                      $665,000

A NASDAQ security is bid at $30.25 and offered at $30.75. An over-the-counter trader effects a trade at $30.75 and charges a commission of $.50 to the customer. The price that will show on the tape is:

Answers

Answer:

$30.75

Explanation:

Given that

Security bidding = $30.25

Offered price = $30.75

over the counter trading = $30.75

Commission charged = $0.50

based on the above information, the price that shows on the tape is equivalent to the over the counter trading price i.e $30.75 also it does not include the commission charged i.e $0.50

Hence, the price is $30.75

Part-time workers likely result in A. inaccurately high estimates of the labor force. B. inaccurately low estimates of the labor force. C. a disincentive for the unemployed to seek employment. D. lower incomes and fewer jobs.

Answers

Answer:

Correct answer:

A. inaccurately high estimates of the labor force.

Explanation:

Part-time work is the type of work where an individual has a flexible work plan is a given company unlike the traditional full-time work. Doing such work create the impression that, there is high labour force among the various industries and sectors. For example, someone might be working in two different firms under part-time basis same day which create an impression of two different individuals.

Deming, the proponent of total quality management, argued that management has the responsibility to train employees in new skills.
A. True
B. False

Answers

Answer:

Its TRUE  

Explanation:

Management should train employees in new skill, where Deming argued that management has the responsibility to train employees in new skills to keep pace with changes in the workplace. In addition, he believed that achieving better quality requires the commitment of everyone in the company.

The last dividend paid by Coppard Inc. was $1.25. The dividend growth rate is expected to be constant at 27.5% for 3 years, after which dividends are expected to grow at a rate of 6% forever. If the firm's required return (rs) is 11%, what is its current stock price

Answers

Answer:

36.38

Explanation:

The Current stock price can be calculated by identifying Present value of dividends in all three years adding terminal value of dividends in year 3.

Year Dividend Growth  Dividend   PV factor  Present Values

1  1.25           127.5%   1.59    0.900901         1.43  

 2  1.59           127.5%   2.03           0.811622          1.64  

 3  2.03          127.5%   2.59    0.731191     1.88  

 3                                    42.987(w)  0.731191           31.43  

Total PV                                                                     36.38  

Current Dividend = 2.59    

Rate of return       = 11.00%    

Growth Rate        = 6.00%    

Terminal value = Current Dividend*(1+Growth rate)/(Rate of return-Growth Rate)

Terminal value = 2.59 x (1+0.06) / (0.11-0.06)  

Terminal value =42.987

   

Current stock price = 1.43 +1.64+1.88+31.43

Current stock price = 36.38  

"Which of the following are covered under the Securities Exchange Act of 1934? I Registration of new issues II Stabilization of new issues III Registration of exchanges IV Registration of broker/dealers"

Answers

Answer: II. stabilization of new issues

III. registration of exchanges

IV. registration of broker-dealers

Explanation:

The Securities Exchange Act of 1934 was put in place in order to be in charge of security trading.

From the options, those that are covered under the Securities Exchange Act of 1934 include the stabilization of new issues, the registration of exchanges and the registration of broker/dealers.

It should be noted that the Securities Exchange Act of 1934 does not cover the registration of new issues.

Slack Inc. borrowed $400,000 on April 1. The note requires interest at 12% and principal to be paid in one year. How much interest is recognized for the period from April 1 to December 31? a. $0. b. $48,000. c. $32,000. d. $36,000

Answers

Answer:

D.$36,000

Explanation:

Calculation for How much interest is recognized for the period from April 1 to December 31

First step is to find the 12% of the amount that was borrowed which is $400,000

$400,000×12%

=$48,000

Now let calculate for the amount of interest that is recognized from April 1 to December 31

Interest =$48,000×3/12

Interest =$12,000

Hence,

Interest =$48,000-$12,000

Interest=$36,000

Therefore the amount of interest that is recognized from April 1 to December 31 will be $36,000

You have gathered the following information on your investments. What is the expected return on the portfolio? Stock Number of Shares Price per Share Expected Return F 310 $ 40 13.32 % G 315 $ 26 10.05 % H 255 $ 52 10.59 %

Answers

Answer:

Expected return on the portfolio = $3,879.00

Explanation:

a) Data and Calculations:

Stock   Number of Shares   Price per Share  Expected Return  Expected

                                                                                                       Value

F                   310                         $ 40                   13.32 %           $1,651.68

G                  315                         $ 26                   10.05 %            $823.09

H                 255                         $ 52                   10.59 %          $1,404.23

Total           880                                                                          $3,879.00

b) The expected return on the portfolio is the addition of the expected returns of each class of shares.  This is obtained by multiplying the number of shares in each class with the price and the expected return in percentage.  This gives a weighted value for the class of shares, which are then added to obtain the expected return on the portfolio.

Seven Manufacturing Corporation uses both standards and budgets. The company estimates that production for the year will be 100,000 units of Product Fast. To produce these units of Product Fast, the company expects to spend $600,000 for materials and $800,000 for labor.

Required:
Compute the estimates for a standard cost.

Answers

Answer:

Unitary cost= $14

Explanation:

Giving the following information:

Production= 100,000

To produce these units of Product Fast, the company expects to spend $600,000 for materials and $800,000 for labor.

First, we need to calculate the total cost and then the unitary cost:

Total cost= 600,000 + 800,000= $1,400,000

Unitary cost= 1,400,000/100,000= $14

The Securities and Exchange Commission requires companies listing on the New York Stock Exchange and the Nasdaq Stock Market to have codes of ethics. A code of ethics is

Answers

Answer:

A Code of Ethics are a set of guidelines that helps the member in distinguishing right and wrong and always following the guidelines that protects the interest of profession and stakeholders.

Explanation:

Basically these Ethical codes are set of guidelines that helps the entities and professionals to acknowledge what is expected from them and what are their responsibilities. Usually every reputable profession and organizations adopt code of ethics to encourage and enforce ethical practices in decision making process.

Answer:

Answer:

A Code of Ethics are a set of guidelines that helps the member in distinguishing right and wrong and always following the guidelines that protects the interest of profession and stakeholders.

Explanation:

Basically these Ethical codes are set of guidelines that helps the entities and professionals to acknowledge what is expected from them and what are their responsibilities. Usually every reputable profession and organizations adopt code of ethics to encourage and enforce ethical practices in decision making process.

Explanation:

of a portfolio. The beta of four stocks​G, ​H, I, and Jare ​, ​, ​, and ​, respectively. What is the beta of a portfolio with the following weights in each​ asset: LOADING...​? What is the beta of portfolio​ 1?

Answers

Answer: 1.02

Explanation:

The Portfolio Beta will be the weighted average of the betas of the individual stocks in Portfolio 1.

Portfolio Beta = (weight in G * beta of G) + (weight in H * beta of H) + (weight in I * beta of I) + (weight in J * beta of J)

= (0.25 * 0.45) + ( 0.25 * 0.82) + ( 0.25 * 1.14) + ( 0.25 * 1.66)

= 0.1125 + 0.205 + 0.285 + 0.415

= 1.0175‬

= 1.02

Henry​ Crouch's law office has traditionally ordered ink refills 50 units at a time. The firm estimates that carrying cost is 35​% of the ​$12 unit cost and that annual demand is about 235 units per year. The assumptions of the basic EOQ model are thought to apply. For what value of ordering cost would its action be​ optimal?

Answers

Answer:

ordering costs = $22.34

Explanation:

economic order quantity (EOQ) = √(2SD / H)

D = annual demand = 235H = holding cost = 35% x $12 = $4.20S = cost per order = ?EOQ = 50

50 = √[(2 x S x 235) / $4.20]

2,500 = (2 x S x 235) / $4.20

$10,500 = 2 x S x 235

S = $10,500 / (2 x 235) = $10,500 / 470 = $22.34

An investor holds a 10 year bond pays a coupon rate of 9%. The yeid to maturity of the bond is 10% . The bond is trading:

Answers

Answer:

the bond is trading at a discount

Explanation:

There is an inverse relationship between the yield and the price of the bond.

As the yield goes up, the price of the bond goes down and as the yield goes down, the price of the bond goes up.

The yield - 10%, is greater than the coupon rate - 9%, the price will be less than the par value, and we say that the bond is trading at a discount.

In Macroland autonomous consumption equals 100, the marginal propensity to consume equals 0.75, net taxes are fixed at 40, planned investment is fixed at 50, government purchases are fixed at 150, and net exports are fixed at 20. Planned aggregate expenditure equals:________a.1,000. b.1,160. c.1,280. d.1,440.

Answers

Answer:

b) $1,160

Explanation:

From the above information,

I=Investment = 50

G=Government expenditure = 150

X=Net export = 20

a=autonomous consumption = 100

b=Marginal propensity to consume = 0.75

Y=Equilibrium GDP

C = consumption ;

C = 100 + 0.75Y (Y income - 40 taxes)

Planned aggregate expenditure (PAE)

PAE = C + l +G +X

Substituting for C in the above equation,

PAE = 100 + 0.75 (Y - 40) + 50 + 150+ 20

= 100 + 0.75Y -30 + 50 + 150 + 20

= 290 + 0.75Y

Since short run exists when Y = PAE

Therefore,

Y = 290 + 0.75Y

Collect like terms

Y - 0.75Y = 290

0.25Y =290

Y = 290/0.25

Y = 1,160

​Keith, an employee of​ Sunbeam, Inc., has gross salary for May of​ $15,000. The entire amount is under the OASDI limit of​ $118,500 and thus subject to FICA. He is also subject to federal income tax at a rate of​ 20%. Which of the following is a part of the journal entry to record the disbursement of his net​ pay? (Assume a FICAOASDI Tax of​ 6.2% and FICAMedicare Tax of​ 1.45%.) (Round the final answer to the nearest​ dollar.)

Answers

Answer:

there are no options listed, but the journal entry to record Keith's salary should be:

May 31, wages expense

Dr Wages expense 15,000

Dr FICA taxes expense 1,147.50

Dr FUTA taxes expense 900

    Cr Federal income taxes withheld payable 3,000

    Cr FICA OASDI taxes withheld payable 930

    Cr FICA Medicare taxes withheld payable 217.50

    Cr FICA OASDI taxes payable 930

    Cr FICA Medicare taxes payable 217.50

    Cr Wages payable 10,852.50

I didn't include SUTA taxes or any other discount (e.g. health insurance, IRA contributions, union contributions, etc.) because sometimes they do not exist, but the previous ones always exist.

Alpha Industries is considering a project with an initial cost of $9.1 million. The project will produce cash inflows of $1.84 million per year for 7 years. The project has the same risk as the firm. The firm has a pretax cost of debt of 5.94 percent and a cost of equity of 11.49 percent. The debt–equity ratio is .71 and the tax rate is 40 percent. What is the net present value of the project?

Answers

Answer:

NPV = $1.22  million

Explanation:

The Net present value (NPV) is the difference between the Present value (PV) of cash inflows and the PV of cash outflows. A positive NPV implies a good investment decision and a negative figure implies the opposite.  

NPV of an investment:  

NPV = PV of Cash inflows - PV of cash outflow  

To work oit the NPV we would need to determine the discount rate i.e cost of capital as follows:

Cost of capital -discount rate -

WACC = We×Ke + Wd×Kd

After cost o debt = 5.94× (1-0.4)=3.56

WACC = (0.71×3.56 %)  + (0.29×11.49%)=5.86 %

PV of cash inflow = A× (1- (1+r)^(-n))/r

A- annul cash inflow, r- 5.86%, n- 7

PV of cash inflow= 1.84 million × (1- 1.0586^(-7))/0.0586 =10.32

Initial cost = 9.1 million

NPV = 10.32  - 9.1 =  1.22 million

NPV = $1.22  million

Margin on price as a percentage is the expression of how much you mark your product up by to arrive at your retail price. True False

Answers

Answer:

False

Explanation:

The margin on price refers to a percentage by taking a difference between the gross profit and the selling price

Here gross profit comes by

= Selling price - cost price

Now in the cost price we added some markup percentage i.e most probably equivalent to the retail price

Hence, the given statement is false

Wyckam Manufacturing Inc. has provided the following information concerning its manufacturing costs:
Fixed Cost per Month Cost per Machine-Hour
Direct materials $ 5.40
Direct labor $ 42,400
Supplies $ 0.30
Utilities $ 1,700 $ 0.25
Depreciation $ 15,200
Insurance $ 11,600
For example, utilities should be $1,700 per month plus $0.25 per machine-hour. The company expects to work 4,200 machine-hours in June. Note that the company’s direct labor is a fixed cost.
Required:
Prepare the company's planning budget for manufacturing costs for June.

Answers

Answer:

Total Manufacturing Costs is $95,680

Explanation:

                        Wyckam Manufacturing Inc.

              Planning Budget for Manufacturing costs

                       For the month Ended June 30

Direct Materials      (4,200 hours *$5.40)                    $22,680

Direct Labor                  Fixed                                        $42,400

Supplies                  (4,200 hours * $0.25 )                   $1,050

Utilities                   ($1,700+ 4,200 Hours * $0.25)      $2,750

Depreciation                  Fixed                                        $15,200

Insurance                       Fixed                                        $11,600

Total Manufacturing Costs                                         $95,680

As the workforce becomes more diverse, why does performance appraisal become a more difficult process?

Answers

Answer:

Performance appraisal in a company with diverse workforce becomes difficult because of some cultural biases that may exist between the manager, who is doing the appraisal, and the diverse workforce.  This problem becomes more acute if the manager is culturally biased and discriminatory by practise.

Explanation:

Company A can have a diverse workforce if it is made up of employees from culturally different places working together in the same workplace.  Bias often arises due to human cultural nuisances.  This becomes more obvious where managers are from some particular cultures while the employees are from mixed cultures.  In such situations, the managers need to be retrained to enable them embrace cultural diversity in the workplace and in performance evaluation.

The key cause due to which the performance appraisal becomes problematic due to diversity in the workforce would be:

- Cultural bias

What is performance appraisal?

Performance appraisal is described as the process of reviewing the performances done by the employees in a particular organization to attain its goals and reward them accordingly.

When the workforce of a particular company or organization becomes exceedingly diverse, it becomes problematic to do performance appraisals.

The reason behind this is that this diversity gives rise to cultural biases and may result in discrimination.

Learn more about "Performance" here:

brainly.com/question/22735387

Your textbook discussed a model of a simple economy with four markets: labor, capital, energy, and food. Which of the following statements is inconsistent with a general equilibrium for this simple economy?
A. The household demand for energy equals the industry supply of energy.
B. The household demand for food equals the industry supply of food.
C. The household demand for labor equals the industry supply of labor.
D. The household supply of capital equals the industry demand for capital.

Answers

Answer:

The correct answer is the option C: The household demand for labor equals the industry supply of labor

Explanation:

To begin with, when it comes to the microeconomics theory the market of labor is considered to be as a factor of production market and from that point of view the labor is demanded by the companies to the households who are the ones who offered the labor due to the fact that the workers are the one who put their force to disposition of the companies. And that is why that it would be inconsistent to say that the household demand for labor will equals the industry supply of labor, because it is all the way around, the household supply of labor will equals the industry demand of it.

Which of the following statements is not true about self-awareness?

a. Self-awareness involves a capacity to monitor and control biases that potentially affect your decision making.
b. Managers who have low self-awareness are superior performers.
c. Self-awareness can be increased by acquiring multiple experiences in diverse situations and with diverse others.
d. Self-awareness is best described as the capacity for introspection and the ability to reconcile oneself as an individual separates from the environment and other individuals.

Answers

Answer:

b. Managers who have low self-awareness are superior performers.

Explanation:

Self-awareness can be defined as a mental process, which occurs when an individual has knowledge about himself, about his knowledge, his actions and attitudes.

Therefore, in the workplace, having self-awareness is essential for a manager to achieve high performance, as this is a skill that includes knowing your skills, values, internal resources that ensure that there is the possibility of better monitoring of environments and oneself, control of emotions and improved perception of oneself and others.

This is a characteristic that adds to a manager 's assertive ability to establish communication focused on ethics, mutual respect, cordiality, etc.

The statement that is not true about self awareness from the list is B. "Managers who have low self-awareness are superior performers."

Self awareness refers to the ability of one to understand their thoughts, feelings, impulses and actions. Being self aware is a very important leadership attribute.

Therefore the statement that 'managers who have low self awareness are superior performers' is wrong.

A manager that lacks self awareness will definitely not perform their job effectively.

Self awareness helps one to become better at making decisions, it helps control biases, communicate more effectively and build good relationships in the work place or other places.

Thus, we can conclude that managers who have low self awareness are not superior performers.

Learn more about self awareness here https://brainly.com/question/2071859?referrer=searchResults

A stock had returns of 17.88 percent, −5.16 percent, and 20.39 percent for the past three years. What is the variance of the returns?

Answers

Answer:

Variance of the return = 0.01983

Explanation:

[tex]S^{2}[/tex]= Σ[tex](X-X)^{2}[/tex]/ N - 1

Mean return = 17.88% + -5.16% + 20.39% = 11.0367%  

Variance = [(17.88% - 11.0367%)2 + (-5.16% - 11.0367%)2 + (20.39% - 11.0367%)2] /(3 - 1)

Variance = [0.004683 + 0.026233 + 0.008748]/2

Variance = 0.01983

Click to review the online content. Then answer the question(s) below, using complete sentences. Scroll down to view additional questions. Career Connection: Shin-fong How does Shin-fong keep track of his finances?

Answers

Answer:

By means of a budget he prepared.

Explanation:

According to the information available, Shing-fong has a carefully thought out strategy. Here's some of what he does;

he keeps tracks of his finances by means of a budget plan.he views all his transactions also checking his debit or credit cards to keep track of how much he spendsShing-Fong avoids eating out as much as he used to and preparing cheaper food at home.he also avoids unnecessarily spending with friends whenever he is invited.

The Clifford Corporation has announced a rights offer to raise $17 million for a new journal, the Journal of Financial Excess. This journal will review potential articles after the author pays a nonrefundable reviewing fee of $6,000 per page. The stock currently sells for $42 per share, and there are 2.9 million shares outstanding. a. What is the maximum possible subscription price? What is the minimum? (Leave no cells blank - be certain to enter "0" wherever required.) b. If the subscription price is set at $34 per share, how many shares must be sold? How many rights will it take to buy one share? (Do not round intermediate calculations. Round your rights needed answer to 2 decimal places, e.g., 32.16.) c. What is the ex-rights price? What is the value of a right? (Do not round intermediate calculations and round your answers to 2 decimal places, e.g., 32.16.) d. A shareholder with 2,000 shares before the offering has no desire (or money) to buy additional shares offered as rights. What is his portfolio value before and after the rights offer? (Do not round intermediate calculations and round your answers to nearest whole number, e.g., 32.)

Answers

Answer:

A.Maximum possible subscription price $42 per shares

Minimum price $0

B.Number of new shares $500,000

Numbers of right needed 5.8

C.Ex-rights price $40.82

Value of a right $1.18

D.Portfolio value before the right offer $84,000

Portfolio value after the right offer $84,000

Explanation:

A.

The maximum possible subscription price based on the information given will be $42 per Shares

The minimum price will be anything that is greater or higher that $0

B. Calculation for how many shares must be sold

Using this formula

Number of new shares =Journal of Financial Excess amount /Subscription price per share

Let plug in the formula

Number of new shares=$17,000,000/ $34 per share

Number of new shares=$500,000

Calculation for how many rights will it take to buy one share

Using this formula

Numbers of right needed=Shares Outstanding/Number of new Shares

Let plug in the formula

Numbers of right needed=$2,900,000/$500,000

Numbers of right needed=5.8

C. Calculation for the ex-rights price

Using this formula

Ex-rights price=(Numbers of right needed*Maximum possible subscription price +Subscription price per share)/(Numbers of right needed+ One shares)

Let plug in the formula

Ex-rights price=(5.8*$42+$34)/(5.8+1)

Ex-rights price=$277.6/6.8

Ex-rights price=$40.82

Calculation for the value of a right

Using this formula

Value of a right =maximum possible subscription price-Ex-rights price

Let plug in the formula

Value of a right=$42-$40.82

Value of a right=$1.18

D. Calculation for What is his portfolio value before the right offer

Using this formula

Portfolio value before the right offer= Shareholders Shares *Maximum possible subscription price

Let plug in the formula

Portfolio value before the right offer=2,000*42

Portfolio value before the right offer=$84,000

Calculation for What is his portfolio value after the right offer

Using this formula

Portfolio value after the right offer=(Shareholders Shares*Ex-rights price) +(Shareholders Shares*Value of a right)

Let plug in the formula

Portfolio value after the right offer=(2,000*40.82)+(2,000*1.18)

Portfolio value after the right offer=$81,640+$2,360

Portfolio value after the right offer=$84,000

g An increase in taxes when the economy is above full employment ​ ______ aggregate demand and real​ GDP, and the price level​ ______.

Answers

Answer:

C.  ​decreases; falls

Explanation:

As we know that

The rise in taxes results in low disposable income for individuals that lowered the spending of the consumer also the consumer spending is an element of the aggregate demand so ultimately it declines that result the curve to shift leftward or downward

Due to this, the real GDP also falls, and the price level too

Hence, the correct option is c.

Other Questions
If a historian wanted to determine wether iron tools could be found in the ruins of an ancient African city, which kind of expert should he consult The size of a television screen is given as 95 cm, correct to the nearest 5 cm.Write down the upper bound of the size of the television screen. Blossom, Inc., manufactures golf clubs in three models. For the year, the Big Bart line has a net loss of $4,700 from sales $201,000, variable costs $175,000, and fixed costs $30,700. If the Big Bart line is eliminated, $19,800 of fixed costs will remain. Prepare an analysis showing whether the Big Bart line should be eliminated. (Enter negative amounts using either a negative sign preceding the number e.g. -45 or parentheses e.g. (45).) g Can you help me find all the seventh roots of unity? what do they look like graphed? (06.03 LC) Which of the following is an example of how HIV can be transmitted from one person to another? Contact between infected blood and a mucus membrane Contact between infected saliva and an open wound Contact between infected blood and skin Contact between infected saliva and lining of mouth Find the intersection point for the following liner function f(x)= 2x+3 g(x)=-4x-27 Is 100 a good estimate for the difference of 712 and 589? If it is, explain why it is a good estimate. If it is not, explain why it is a bad estimate. Using the insurance company's assumptions, calculate the probability that there are fewer than 3 tornadoes in a 14-year period. Round your answer to four decimal places. Margarita, age 30, used to work 80 hours a week as a corporate lawyer. this year she quit her job to take care of her five-month-old daughter. she is currently not looking for a job. the survey conducted by the bureau of labor statistics will count margarita today as choose one:a. not in the labor force.b. not in the working-age population.c. a discouraged worker.d. employed.e. unemployed. Maya can't find her red sandals. Tamiko is wearing red sandals. Therefore, Tamiko stole Maya's shoes. This is an example of When the body cells are hypotonic to the blood plasma, water will move from intracellular fluid to extracellular fluid.A. TrueB. False Consider these metal ion/metal standard reduction potentials Cd2+(aq)|Cd(s) Zn2+(aq)|Zn(s) Ni2+(aq)|Ni(s) Cu2+(aq)|Cu(s) Ag+(aq)|Ag(s) -0.40 V -0.76 V 0.25 V +0.34 V +0.80 V Based on the data above, which species is the best reducing agent? ANSWER QUICKLY PLZZZZZZ ASAP READ QUESTIONS CAREFULLY in a polynomial function of degree 5, what is the maximum number of extreme that could be possible? (please explain with the answer if possible!) The time between consecutive uses of a vending machine is exponential with an average of 15 minutes. a)Given that the machine has not been used in the previous 5 minutes, what is the probability that the machine will not be used during the next 10 minutes help me plz can somewon help me You find a zero coupon bond with a par value of $10,000 and 14 years to maturity. The yield to maturity on this bond is 5.1 percent. Assume semiannual compounding periods. What is the price of the bond What were Malcolm's and Ravi's maximum speeds? Scenario B You are a principal who is trying to figure out the truth about a lunchroom fight. The fight was between Justin and Max. Justin is a new student. He is shy and doesnt have many friends. Max is a popular student who is known for his friendliness. Account A: Justin Max started it. I was just standing in line waiting to pay for my food, and he shoved me super hard. And for no reason! He just freaked out on me. I dont even know the kid, and hes been weird to me ever since I started going to this school. Him and his friends glare at me in English class for no reason. Account B: Max That kid is psycho. He turned around and punched me out of nowhere. Me and my friends were standing in line just joking around, and he turned around and punched me for no reason. Hes messed up and creepy. Ask anyone. Account C: Jamie (student who has class with Max and Justin right before lunch) I wasnt in the cafeteria today, and Im not friends with any of those guys, but Ive seen Max and his friends be mean to Justin in the hallways and in class when the teacher isnt looking. Not physical or anything, but theyll like say jokes under their breath and then laugh and stuff like that. They make him uncomfortable." Which of the following is equivalent to 2i(6 7i)?